2025 AMC 10A Problems/Problem 5: Difference between revisions
No edit summary |
|||
| Line 93: | Line 93: | ||
-Kasoisanti | -Kasoisanti | ||
==Chinese Video Solution== | |||
https://www.bilibili.com/video/BV1gV2uBbEJe/ | |||
~metrixgo | |||
==Video Solution== | ==Video Solution== | ||
Revision as of 01:45, 7 November 2025
Consider the sequence of positive integers
What is the
th term in the sequence?
Video Solution
Solution 1
One possible way the sequence could've been constructed was by putting "mountains" going up from
, to
then going back down to
For example, the first few "mountains" look like this:
So, the
mountain has length
and has highest number
We want to add mountains until we get a total length as close as possible, but not exceeding,
Let the last mountain we sum be mountain
Hence,
so our max
is
In this
mountain, the max number is
so the
mountain has max number
Next,
so we're looking for the
number in the
mountain, which is
~Tacos_are_yummy_1
- This problem and problem 11 should've switched places tbh ;-; ~tacos
- FRR
Video Solution (Fast and easy)
https://youtu.be/ZQhAdIs2FIg?si=mEgnYvdQ2sMrCx1P ~ Pi Academy
Solution 2 (nice equation)
Group the numbers by their hill pattern:
The maximums of each hill occur at terms
These terms correspond to maximums of
Let
be the maximum at term
. Since the sum of the first
odd numbers is
we have
So for example, if a =
then
telling us that the peak of the hill with maximum
occurs at the
th term.
Now, we know
so let
Then
so the
th term is
Then the
th term must be
Solution 3
Note that the first
is the first number from the left, the second
is the third number from the left, and the third
is the seventh number from the left. Since the second differences between the indices of the
th
’s are equal, there is a quadratic function for the position of the
th
. As the second differences are
, the leading coefficient of this function is
. Let the function be
. Then, we have
, and
. Solving, we get
and
. Therefore, the
th
from the left is the
th number from the left. Plugging in
gives
, so the
st number is a
.
Also, the sequence shows that the first instance of a positive integer
is always to the left of the
th
. This means that following the
st number, the numbers go to or above
. Adding
to
means that the
th term of the sequence is
~joiceeliu
Solution 4 (Quick and Fast!🚀)
Note that after every round, the number of terms is the highest term squared. For example, after the threes, there are
terms. We also notice that
, so the
th term is
, which is answer choice
~iiiiiizh
Solution 5 (Super Simple!!)
Upon inspecting the problem, we see that the 1st term is 1, and so are the 3rd, 7th, 13th, 18th, 31st, and so on. We see that the pattern comes from the differences: +2,+4,+6,+8,+10, and so on. From this we see that a 1 appears at each position that is a number of the form n^2-n+1. We know that 2025 = 45*45. Plugging 45 as n gives 1981, meaning that the 2025th term is 2025-1981+1=45. Therefore, the solution is
~vgarg
Solution 6 - ⚡ Very Fast! - Basic Math (& Pattern Recognition)
Immediately by looking at the sequence, we observe that the digit in the place of each perfect square is the square root of that number. For example, the first term is \(1\) (because \(\sqrt{1}=1\)), the \(4^{th}\) term is \(2\) (because \(\sqrt{4}=2\)), the \(9^{th}\) term is \(3\) (because \(\sqrt{9}=3\)), the \(16^{th}\) term is \(4\) (because \(\sqrt{16}=4\)), and so on. And since \(2025\) is a perfect square, it makes it easy for us because \(\sqrt{2025}=45\), which makes the answer choice \(\boxed{\textbf{(E) }45}.\)
To make it easier to understand my way of solving it, here is a table.
\(
\begin{array}{c|c|c}
\text{n} & \text{
} \\[6pt]
1 & 1 \\[6pt]
4 & 2 \\[6pt]
9 & 3 \\[6pt]
16 & 4 \\[6pt]
25 & 5 \\[6pt]
36 & 6 \\[6pt]
{...} & {...} \\[6pt]
2025 & \boxed{\textbf{(E) }45}
\end{array}
\)
~i_am_not_suk_at_math (saharshdevaraju 21:14, 6 November 2025 (EST)saharshdevaraju)
Solution 7 - Pattern Recognition
After looking at the problem, it is evident that a pattern exists for the first occurence of a number in the sequence. Knowing this, we can write the places that each number appears at:
\begin{align}
&1: 1 \\
&2: 2 \\
&3: 5 \\
&4: 10 \\
&5: 17
\end{align}
If we write down the difference between the each numbers position in sequence, we get
. We notice how these numbers are similar to how the square numbers increase (square numbers:
). We also notice how the position of the first instance of each number
is just
. Since we're trying to find the
th number and
, we can plug in
for
. As a result, we get that the first instance of the number
appears at
or
. Knowing that the number before the first instance of a number
is
, we get the answer choice
-Kasoisanti
Chinese Video Solution
https://www.bilibili.com/video/BV1gV2uBbEJe/
~metrixgo
Video Solution
~MK
Video Solution by Daily Dose of Math
~Thesmartgreekmathdude
See Also
| 2025 AMC 10A (Problems • Answer Key • Resources) | ||
| Preceded by Problem 4 |
Followed by Problem 6 | |
| 1 • 2 • 3 • 4 • 5 • 6 • 7 • 8 • 9 • 10 • 11 • 12 • 13 • 14 • 15 • 16 • 17 • 18 • 19 • 20 • 21 • 22 • 23 • 24 • 25 | ||
| All AMC 10 Problems and Solutions | ||
These problems are copyrighted © by the Mathematical Association of America.